Sie sind auf Seite 1von 12

Problem 4.14 The moment exerted about point E by the weight is 299 in-lb.

What moment does the weight exert about point S?


S 30 E
in.

13

in.

12

40

Solution:

The key is the geometry

From trigonometry, cos 40 = d2 d1 , cos 30 = 13 in 12 in

S 30

13

in.

Thus d1 = (12 in) cos 30 d1 = 10.39 and d2 = (13 in) cos 40 d2 = 9.96 We are given that 299 in-lb = d2 W = 9.96 W W = 30.0 lb

12

E
in.

40

d1 S
30

Now, Ms = (d1 + d2 )W Ms = (20.35)(30.0) Ms = 611 in-lb clockwise

13 in

12 i

n
E

40
d2

Problem 4.15 Three forces act on the square plate. Determine the sum of the moments of the forces (a) about A, (b) about B, (c) about C.
200 N C 200 N

Solution: Determine the perpendicular distance between the points and the lines of action. Determine sign, and calculate moment. (a) The distances from point A to the lines of action is zero, hence the moment about A is MA = 0. (b) The perpendicular distances of the lines of action from B are: 3 m for the force through A, with a positive action, and for the force through C, DC = 1 32 + 32 = 2.12 m with a negative action. The moment about B is 2 MB = (3)(200) 2.12(200) = 175.74 N-m (c) The distance of the force through A from C is 3 m, with a positive action, and the distance of the force through B from C is 3 m, with a positive action. The moment about C is MC = 2(3)(200) = 1200 N-m.
200 N

3m

B A 3m 200 N 3m

200 N

F A 3m

200 N B

Problem 4.16 Determine the sum of moments of the three forces about (a) point A, (b) point B, (c) point C.
100 lb A B 200 lb 100 lb C

2 ft

2 ft

2 ft

2 ft

Solution:
(a) The sum of the moments about A:

200 lb 100 lb 100 lb B C

MA = (2)(100) + (4)(200) (6)(100) = 0. (b) The sum of the moments about B:

2 ft
MB = +(2)(100) (2)(100) = 0 (c) The sum of the moments about C: MC = +(6)(100) (4)(200) + (2)(100) = 0.

2 ft

2 ft

2 ft

Problem 4.17 Determine the sum of the moments of the ve forces acting on the Howe truss about point A.
600 lb 400 lb C B A H 4 ft 4 ft I

800 lb 600 lb D 400 lb E 8 ft F G J 4 ft 4 ft K 4 ft L 4 ft

Solution: All of the moments about A are clockwise (negative). The equation for the sum of the moments about A in units of ft-lb is given by:
MA = 4(400) 8(600) 12(800) 16(600) 20(400)

800 lb 600 lb 400 lb C D E F G H 4 ft 4 ft I 4 ft J 4 ft K 4 ft L 4 ft 8 ft 600 lb 400 lb

or

MA = 33,600 ft-lb.

B A

Problem 4.22 The vector sum of the three forces is zero, and the sum of the moments of the three forces about A is zero. (a) What are FA and FB ? (b) What is the sum of the moments of the three forces about B? Solution:
The forces are: FA = |FA |(0i + 1j), FB = |FB |(0i + 1j), and F = 80(0i 1j). The sum of the forces is: F = FA + FB + F = 0, from which FY = (|FA | + |FB | 80)j = 0. The sum of the moments: MA = (0.9)(80) + (1.3)(|FB |) = 0. (a) Solve these two equations to obtain: |FB | = 55.4 N, and |FA | = 24.6 N (b) The moments about B: MB = (80)(0.4) (1.3)|FA | = 0

80 N A F A 900 mm 400 mm B F B

Problem 4.23 The weights (in ounces) of sh A, B, and C are 2.7, 8.1, and 2.1, respectively. The sum of the moments due to the weights of the sh about the point where the mobile is attached to the ceiling is zero. What is the weight of sh D?

12 in A 6 in 7 in D 2 in C

3 in 2 in B

Solution:
MO = (12)(2.7) 3(10.2 + D) Solving D = 0.6 oz

12 in A 6 in 7 in D 2 in C

3 in 2 in B

12 0

(8.1 + 2.1 +D) = (10.2 + D) 2,7

Problem 4.28 Five forces act on a link in the gearshifting mechanism of a lawn mower. The vector sum of the ve forces on the bar is zero. The sum of their moments about the point where the forces Ax and Ay act is zero. (a) Determine the forces Ax , Ay , and B. (b) Determine the sum of the moments of the forces about the point where the force B acts.
The strategy is to resolve the forces into x- and y-components, determine the perpendicular distances from B to the line of action, determine the sign of the action, and compute the moments. The angles are measured counterclockwise from the x-axis. The forces are F2 = 30(i cos 135 + j sin 135 ) = 21.21i + 21.21j F1 = 25(i cos 20 + j sin 20 ) = 23.50i + 8.55j. (a) The sum of the forces is F = A + B + F1 + F2 = 0. Substituting: FX and FY = (AX + BX + 23.5 21.2)i = 0, = (AY + 21.2 + 8.55)j = 0.

Ay Ax 25 kN 20 650 mm 30 kN 45 B 650 mm 350 mm 450 mm

Solution:

Ay Ax F1 = 25 kN 650 mm F2 = 30 kN 45 B 650 mm 350 mm 20 450 mm

Solve: BX = 20.38 kN. Substitute into the force equation to obtain AX = 18.09 kN (b) The distance from B to the line of action of the y-component of F1 is 0.350 m, and the action is negative. The distance from B to the line of action of AX is 0.650 m and the action is negative. The distance from B to the line of action of AY is 1 m and the action is positive. The distance from B to the line of action of the x-component of F2 is 0.450 m and the action is negative. The sum of the moments about B: MB = (0.350)(21.21) (0.650)(18.09) + (1)(29.76) (0.450)(23.5) = 0

Solve the second equation: AY = 29.76 kN. The distances of the forces from A are: the triangle has equal base and altitude, hence the angle is 45 , so that the line of action of F1 passes through A. The distance to the line of action of B is 0.65 m, with a positive action. The distance to the line of action of the y-component of F2 is (0.650 + 0.350) = 1 m, and the action is positive. The distance to the line of action of the x-component of F2 is (0.650 0.450) = 0.200 m, and the action is positive. The moment about A is MA = (8.55)(1) + (23.5)(0.2) + (BX )(0.65) = 0.

Problem 4.29 Five forces act on a model truss built by a civil engineering student as part of a design project. The dimensions are b = 300 mm and h = 400 mm; F = 100 N. The sum of the moments of the forces about the point where Ax and Ay act is zero. If the weight of the truss is negligible, what is the force B?
Ax Ay b b b

60

60

Solution:

The x- and y-components of the force F are

F = |F|(i cos 60 + j sin 60 ) = |F|(0.5i + 0.866j). The distance from A to the x-component is h and the action is positive. The distances to the y-component are 3b and 5b. The distance to B is 6b. The sum of the moments about A is MA = 2|F|(0.5)(h) 3b|F|(0.866) 5b|F|(0.866) + 6bB = 0. Substitute and solve: B =
1.6784|F| 1.8

60

F 60

h Ax Ay b b b b b b B

= 93.2 N

Problem 4.36 The cable from B to A (the sailboats forestay) exerts a 230-N force at B. The cable from B to C (the backstay) exerts a 660-N force at B. The bottom of the sailboats mast is located at x = 4 m, y = 0. What is the sum of the moments about the bottom of the mast due to the forces exerted at B by the forestay and backstay?

y B (4,13) m

A (0,1.2) m

C (9,1) m

Solution:
tan =

Triangle ABP

4 , = 18.73 11.8

B (4,13) m
Triangle BCQ 5 , = 22.62 12

tan =

+MO = (13)(230) sin (13)(660) sin +MO = 2340 N-m

B (4,13)

A (0,1.2) m 230 N 660 N 230 sin


C (9,1) m

660 sin

P A (0,1.2) C (9,1) Q O (4,0) O 13 m

Problem 4.60 The direction cosines of the force F are cos x = 0.818, cos y = 0.182, and cos z = 0.545. The support of the beam at O will fail if the magnitude of the moment of F about O exceeds 100 kN-m. Determine the magnitude of the largest force F that can safely be applied to the beam. Solution: The strategy is to determine the perpendicular distance from O to the action line of F, and to calculate the largest magnitude of F from MO = D|F|. The position vector from O to the point of application of F is rOF = 3i (m). Resolve the position vector into components parallel and normal to F. The component parallel to F is rP = (rOF eF )eF , where the unit vector eF parallel to F is eF = i cos X + j cos Y + k cos Z = 0.818i + 0.182j 0.545k. The dot product is rOF eF = 2.454. The parallel component is rP = 2.007i + 0.4466j 1.3374k. The component normal to F is rN = rOF rP = (3 2)i 0.4466j + 1.3374k. The magnitude of the normal component is the perpendicular distance: D = 12 + 0.44662 + 1.3372 = 1.7283 m. The maximum moment allowed is MO = 1.7283|F| = 100 kN-m, from which
|F| = 100 kN-m = 57.86 58 kN = 1.7283 m

3m

O 3m

F x

Problem 4.61 The force F exerted on the grip of the exercise machine points in the direction of the unit vector 2 2 e = 3 i 3 j+ 1 k and its magnitude is 120 N. Determine 3 the magnitude of the moment of F about the origin O.

150 mm

Solution:
force is

The vector from O to the point of application of the

O 200 mm z 250 mm x

r = 0.25i + 0.2j 0.15k m and the force is F = |F|e or F = 80i 80j + 40k N.

y
The moment of F about O is i MO = r F = 0.25 80 or MO = 4i 22j 36k N-m and |MO | = 42 + 222 + 362 N-m j 0.2 80 k 0.15 N-m 40

150 mm

F O z 250 mm x 200 mm

|MO | = 42.4 N-m

Problem 4.62 The force F in Problem 4.61 points in the direction of the unit vector e = 2 i 2 j + 1 k. The 3 3 3 support at O will safely support a moment of 560 N-m magnitude. (a) Based on this criterion, what is the largest safe magnitude of F? (b) If the force F may be exerted in any direction, what is its largest safe magnitude? Solution:
See the gure of Problem 4.61. If we set |MO | = 560 N-m, we can solve for |Fmax | 560 = 0.353|Fmax | |Fmax | = 1586 N (b) If F can be in any direction, then the worst case is when r F. The moment in this case is |MO | = |r||Fworst | |r| = 0.252 + 0.22 + 0.152 = 0.3536 m

The moment in Problem 4.61 can be written as i = 0.25 2 F 3 j 0.2 2F 3 k 0.15 where F = |F| 1 +3F

MO

MO = (0.0333i 0.1833j 0.3k)F And the magnitude of MO is |MO | = ( 0.03332 + 0.18332 + 0.32 )F

560 = (0.3536)|FWORST | |Fworst | = 1584 N |MO | = 0.353 F

Problem 4.63 An engineer estimates that under the most adverse expected weather conditions, the total force on the highway sign will be F = 1.4i2.0j (kN). What moment does this force exert about the base O?

y F

CCON TU
8m

8m

O x

Solution: The coordinates of the point of application of the force are: (0, 8, 8). The position vector is rOF = 8j + 8k. The cross product is
i 0 1.4 j k 8 8 = 16i (1.4)(8)j + (1.4)(8)k 2 0

rOF F =

8m

MO = 16i 11.2j 11.2k (N-m) Check: Use perpendicular distances to forces: MX = 8(2) = 16, MY = 8(1.4) = 11.2,

8m

O x

MZ = 8(1.4) = 11.2 .

Problem 4.70 Consider the 70-m tower in Problem 4.69. Suppose that the tension in cable AB is 4 kN, and you want to adjust the tensions in cables AC and AD so that the sum of the moments about the origin O due to the forces exerted by the cables at point A is zero. Determine the tensions. Solution:
From Varignons theorem, the moment is zero only if the resultant of the forces normal to the vector rOA is zero. From Problem 4.69 the unit vectors are: rAD 35 70 35 = i j |rAD | 85.73 85.73 85.73 rAC 40 70 40 = i j+ k |rAC | 90 90 90 The tensions are TAB = 4eAB , TAC = |TAC |eAC , and TAD = |TAD |eAD . The components normal to rOA are FX = (0.4082|TAD | 0.4444|TAC | + 1.9846)i = 0 FZ = (0.4082|TAD | + 0.4444|TAC |)k = 0. The HP-28S calculator was used to solve these equations: |TAC | = 2.23 kN, |TAD | = 2.43 kN

eAD =

= 0.4082i 0.8165j 0.4082k eAC =

= 0.4444i 0.7778j + 0.4444k eAB rAB 40 70 = = i j + 0k = 0.4963i 0.8685j + 0k |rAB | 80.6 80.6

Problem 4.71 The tension in cable AB is 150 N. The tension in cable AC is 100 N. Determine the sum of the moments about D due to the forces exerted on the wall by the cables.
The coordinates of the points A, B, C are A (8, 0, 0), B (0, 4, 5), C (0, 8, 5), D(0, 0, 5). The point A is the intersection of the lines of action of the forces. The position vector DA is rDA = 8i + 0j 5k.

y 5m 5m B C 4m

Solution:

8m
The position vectors AB and AC are

rAB = 8i + 4j 5k, rAC = 8i + 8j + 5k,

rAB = rAC =

82 + 42 + 52 = 10.247 m. 82 + 82 + 52 = 12.369 m.

D z

8m A x

The unit vectors parallel to the cables are: MD = eAB = 0.7807i + 0.3904j 0.4879k, eAC = 0.6468i + 0.6468j 0.4042k. The tensions are TAB = 150eAB = 117.11i + 58.56j 73.19k, TAC = 100eAC = 64.68i + 64.68j 40.42k. The sum of the forces exerted by the wall on A is TA = 181.79i + 123.24j 32.77k. The force exerted on the wall by the cables is TA . The moment about D is MD = rDA TA ,

i 8 181.79

j 0 123.24

k 5 = (123.24)(5)i +32.77

((8)(+32.77) (5)(181.79))j + (8)(123.24)k MD = 616.2i 117.11j 985.9k (N-m) (Note: An alternate method of solution is to express the moment in terms of the sum: MD = (rDC TC + (rDB TB ).)

y 5m

5m

4m

8m z D

A 8m F x

Problem 4.98 The tension in cable AB is 80 lb. What is the moment about the line CD due to the force exerted by the cable on the wall at B?
y 8 ft 3 ft C B

6 ft

A (6, 0, 10) ft

Solution: The strategy is to nd the moment about the point C exerted by the force at B, and then to nd the component of that moment acting along the line CD. The coordinates of the points B, C, D are B (8, 6, 0), C (3, 6, 0), D(3, 0, 0). The position vectors are: rOB = 8i + 6j, rOC = 3i + 6j, rOD = 3i. The vector parallel to CD is rCD = rOD rOC = 6j. The unit vector parallel to CD is eCD = 1j. The vector from point C to B is rCB = rOB rOC = 5i.
The position vector of A is rOA = 6i + 10k. The vector parallel to BA is rBA = rOA rOB = 2i 6j + 10k. The magnitude is |rBA | = 11.832 ft. The unit vector parallel to BA is eBA = 0.1690i 0.5071j + 0.8452k. The tension acting at B is TBA = 80eBA = 13.52i 40.57j + 67.62k. The magnitude of the moment about CD due to the tension acting at B is 0 5 13.52 1 0 40.57 0 0 67.62

|MCD | = eCD (rCB TBA ) = = 338.1 (ft lb).

The moment about CD is MCD = 338.1eCD = 338.1j (ft lb). The sense of the moment is along the curled ngers of the right hand when the thumb is parallel to CD, pointing toward D.

y 3 ft C

8 ft

B 6 ft D x

(6, 0, 10)

Problem 4.119 Four forces and a couple act on the beam. The vector sum of the forces is zero, and the sum of the moments about the left end of the beam is zero. What are the forces Ax , Ay , and B? Solution:
The sum of the forces about the y-axis is

y 800 N Ax Ay x 4m 4m 3m 200 N-m B

FX = AY + B 800 = 0. The sum of the forces about the x-axis is FX = AX = 0. The sum of the moments about the left end of the beam is ML = 11B 8(800) 200 = 0. From the moments: B = 6600 = 600 N. 11

y 800 N Ax Ay 200 N-m B x 4m 4m 3m

Substitute into the forces balance equation to obtain: AY = 800 600 = 200 N

Problem 4.120 The force F = 40i + 24j + 12k (N). (a) What is the moment of the couple? (b) Determine the perpendicular distance between the lines of action of the two forces.

y F (6, 3, 2) m x (10, 0, 1) m z F

Solution:
(a) The moment of the couple is given MC = rAB F MC = (4i + 3j + 1k) (40i + 24j + 12k) MC = 12i + 88j 216k (N-m) (b) |MC | = |d||F| sin 90 |F| = |MC | =
2 2 2 Fx + Fy + Fz = 48.2 N 2 Mx

F B (6, 3, 2) m

x (10, 0, 1) m A z F

2 My

2 Mz

= 233.5 N

|d| = perpendicular distance |d| = |MC |/|F| |d| = 4.85 m

Problem 4.121 Determine the sum of the moments exerted on the plate by the three couples. (The 80-lb forces are contained in the x-z plane.)

y 3 ft 20 lb 3 ft 20 lb 40 lb 8 ft 40 lb x

Solution: The moments of two of the couples can be determined from inspection:
M1 = (3)(20)k = 60k ft lb. M2 = (8)(40)j = 320j ft lb The forces in the 3rd couple are resolved: F = (80)(i sin 60 + k cos 60 ) = 69.282i + 40k

60

80 lb

60

80 lb

3 ft

3 ft 20 lb 20 lb 40 lb x 8 ft

The two forces in the third couple are separated by the vector

40 lb 80 lb z 60 60 80 lb

r3 = (6i + 8k) (8k) = 6i The moment is

The sum of the moments due to the couples: M3 i = r3 F3 = 6 69.282 j k 0 0 = 240j. 0 40 M = 60k + 320j 240j = 80j 60k ft lb

Problem 4.122 What is the magnitude of the sum of the moments exerted on the T -shaped structure by the two couples? Solution:
inspection: The moment of the 50 lb couple can be determined by

3 ft 3 ft 50j (lb) 50i + 20j 10k (lb)

3 ft z 50j (lb) 3 ft 50i 20j + 10k (lb) x

M1 = (50)(3)k = 150k ft lb. The vector separating the other two force is r = 6k. The moment is i 0 50 j 0 20 k 6 = 120i + 300j. 10

M2 = r F =

y 3 ft 3 ft 50 j (lb) 3 ft x

The sum of the moments is M = 120i + 300j 150k. The magnitude is

F
z

50j (lb)

3 ft

|M| =

1202 + 3002 + 1502 = 356.23 ft lb

Problem 4.126

In Problem 4.125, the forces

FB = 2i + 6j + 3k (kN), FC = i 2j + 2k (kN), and the couple MC = MCy j + MCz k (kN-m).

Solution: From the solution to Problem 4.125, the sum of the moments of the two forces about A is
MForces = 0i 7j + 2k (kN-m). The required moment, MC , must be the negative of this sum. MCy = 7 (kN-m), and MCz = 2 (kN-m).

Determine the values for MCy and MCz , so that the sum Thus of the moments of the two forces and the couple about A is zero.

Problem 4.127 Two wrenches are used to tighten an elbow tting. The force F = 10k (lb) on the right wrench is applied at (6, 5, 3) in., and the force F on the left wrench is applied at (4, 5, 3) in. (a) Determine the moment about the x axis due to the force exerted on the right wrench. (b) Determine the moment of the couple formed by the forces exerted on the two wrenches. (c) Based on the results of (a) and (b), explain why two wrenches are used.

Solution: The position vector of the force on the right wrench is rR = 6i 5j 3k. The magnitude of the moment about the x-axis is
1 |MR | = eX (rR F) = 6 0 (a) 0 5 0 0 3 = 50 in lb 10

from which MXL = 50i in lb, which is opposite in direction and equal in magnitude to the moment exerted on the x-axis by the right wrench. The left wrench force is applied 2 in nearer the origin than the right wrench force, hence the moment must be absorbed by the space between, where it is wanted.

The moment about the x-axis is

z x

MR = |MR |eX = 50i (in lb). (b) The moment of the couple is

F F
MC = (rR rL ) FR i = 2 0 j 0 0 k 6 = 20j in lb 10

(c)

The objective is to apply a moment to the elbow relative to connecting pipe, and zero resultant moment to the pipe itself. A resultant moment about the x-axis will affect the joint at the origin. However the use of two wrenches results in a net zero moment about the x-axis the moment is absorbed at the juncture of the elbow and the pipe. This is demonstrated by calculating the moment about the x-axis due to the left wrench: 0 5 0 0 3 = 50 in lb 10

1 |MX | = eX (rL FL ) = 4 0

Das könnte Ihnen auch gefallen